Questions tagged [analytic-number-theory]

A beautiful blending of real/complex analysis with number theory. The study involves distribution of prime numbers and other problems and helps giving asymptotic estimates to these.

Filter by
Sorted by
Tagged with
4 votes
1 answer
152 views

Lattice points on caps on quadrics

Let $(L,q)$ be a non-degenerate quadratic lattice of signature $(n_{+},n_{-})$ and dimension $d\geq 5$. Consider the quadric $C_{k}=\{x\in L_{\mathbb{R}},\, q(x)=k \}$ for some non-zero integer $k$. ...
Yuri's user avatar
  • 41
10 votes
1 answer
296 views

Coefficient bounds on cusp forms, half-integer weight

Let $f(\tau) = \sum_{n=1}^{\infty} a(n) q^n$ be a cusp form on $\Gamma_0(4N)$ of half-integer weight $k \ge 5/2.$ The Ramanujan-Petersson conjecture in this case is that $$a(n) \ll n^{(k-1)/2 + \...
user105068's user avatar
4 votes
1 answer
200 views

The subfield of $\mathbb{C}$ generated by Fourier coefficients of prime index

Let $f=\sum_{n\ge1} a_nq^n$ be an Hecke eigenform cusp form of integral weight $k$ on $\Gamma_0(N)$ with character $\varepsilon\pmod N,$ and let $M_f$ be the subfield of $\mathbb{C}$ generated by the ...
User 101794987's user avatar
0 votes
1 answer
384 views

Diophantine equations and modular forms

Let $D$ be a square-free positive integer which is the fundamental discriminant of a real quadratic field. Consider the following quadratic form $$Q_{D}(x,y)=x^2+Dy^2.$$ My questions are : What is ...
Zakariae.B's user avatar
3 votes
0 answers
142 views

Reference request: Eisenstein series built from principal series representation

One can build an Eisenstein series $E_s(v,g)$ from a vector $v\in D_s$, the principal series representation of $G_{\mathbb{A}_\mathbb{Q}}$. The space $D_s$ has a restricted tensor product structure $$ ...
Ted Mao's user avatar
  • 443
5 votes
1 answer
809 views

Going beyond the Sylvester and Schur theorem with regard to $x,x+1,\dots,x+n-1$

I was recent reading through Paul Erdos's classic elementary proof of Sylvester-Schur. It occurred me that there is a simple argument that when $x$ is sufficiently large and if $p_i$ represents the $...
Larry Freeman's user avatar
4 votes
0 answers
183 views

Does $\pi(x) \geq \mathrm{Li}(x)$ imply that $\vartheta(x) \geq x$

As the question in the title asks, does $\pi(x) \geq \mathrm{Li}(x)$ imply that $\vartheta(x) \geq x$? Here $\pi(x) = \#\{p \leq x\}$, $\vartheta(x) = \sum_{p \leq x} \log p$ and $\mathrm{Li}(x) = \...
Ahmad's user avatar
  • 595
2 votes
1 answer
175 views

Counting prime points in a bounded region

Let $R$ be a semi-algebraic, compact region in $\mathbb{R}^n$ with positive Lebesgue measure. Let $N(R) = \# (R \cap \mathbb{Z}^n)$. Davenport's lemma asserts that we have $$\displaystyle N(R) = \...
Stanley Yao Xiao's user avatar
3 votes
1 answer
492 views

Higher dimensional large sieve inequality

One of the most important achievements in analytic number theory is the establishment of the so-called large sieve inequality, which is formulated as follows. Let $\{a_n\}$ denote a finite sequence of ...
Stanley Yao Xiao's user avatar
2 votes
1 answer
225 views

Upper bound for an exponential sum in Waring-Goldbach problem

In Waring's problem, we have Hua's estimate $$S(a,b,q) = \sum_{x=1}^q e^{2\pi i (ax^k + bx)/q)} \ll q^{1/2+\epsilon} \gcd(b,q),$$ where $(a,q)=1$. ?Do you know a similar upper bound for the sum $$...
Yıldırım A.'s user avatar
3 votes
0 answers
334 views

On discrepancy of integer sequences related to Erdos-Turan-Koksma

Assume we have a sequence $a_1,\dots,a_k\in\Bbb N$ with each $a_i\approx n$ where $n$ is some integer. Suppose there exists an $\eta\in(0,1)$ such that for every $d_1,\dots,d_k\in\Bbb Z$ with $$...
user avatar
-2 votes
1 answer
200 views

Solutions to a diophantine system

What is the smallest $\gamma_1,\gamma_2,\gamma_3>0$ such that given coprime $p,q=\Theta(\ell)$ and integer $t\geq3$ there are coprime $m,n=\Theta(\ell^{t-1})$ with $(mn,pq)=1$, $\alpha_i\in\Bbb Z$ ...
Turbo's user avatar
  • 13.7k
6 votes
1 answer
204 views

How are the ratios of successive values of the divisor function distributed?

One motivation for this question is a paper by Erdos and Hall "Values of the divisor function on short intervals", in which the authors obtain the leading asymptotics $$x(\log x)^{2(\sqrt 2-...
Kevin Smith's user avatar
  • 2,470
0 votes
0 answers
210 views

On the number of $y$-smooth numbers less than a given magnitude

I'm trying to provide a full-fledged proof of the estimate $$\Psi(x,x^{1/u}) = x \, \rho(u)+ O\left(\frac{x}{\log x}\right) \qquad (*)$$ via the sly inductive approach commonly attributed to A. ...
José Hdz. Stgo.'s user avatar
2 votes
0 answers
268 views

Number Theory Problem for 1st/2nd Year Graduate Student

I am looking for a problem in (algebraic) number theory in which a 1st/2nd year graduate student in number theory can make some nontrivial progress. I am not looking for something that can conceivably ...
CL1337's user avatar
  • 21
2 votes
1 answer
164 views

A sieve by not (necessarily) coprime integers

Let $\mathcal{A}$ be a nonempty set of pairwise coprime positive integers, and for each $a \in \mathcal{A}$ let $\Omega_a \subsetneq \{0, \ldots, a - 1\}$ be a set of residues modulo $a$. Furthermore, ...
user avatar
2 votes
1 answer
224 views

Upper bound for a higher dimensional Ramanujan sum

Fix an integer vector $\mathbf m\in \mathbb Z^k$. Let $q$ be a positive integer. Is there a "good" upper bound in terms of $q,\bf m$ for the exponential sum: \[\sum_{\mathbf n} e\left(\frac{\langle m,...
Changguang's user avatar
2 votes
1 answer
285 views

congruence for modular forms coefficients

Let $A$ be the set of modular forms $f=\sum a(n)q^n$ of weight $k$ on $\Gamma_0(N)$ with character $\chi$ whose coefficients $a(n)$ are in the ring of integers $\mathcal{O}$ of a fixed number field $F$...
Zakariae.B's user avatar
8 votes
0 answers
388 views

$L^1$ norm of Fourier transform of subset sums

Let $n_1,\dots,n_k$ be a set of $k$ natural numbers less than $N$, with $k = (1- \delta) \log_2 N$ for $\delta$ relatively small. Let $e(x) = e^{ 2\pi i x}$, as usual. Assume that $$\int_0^1\prod_{j=1}...
Will Sawin's user avatar
  • 137k
16 votes
4 answers
2k views

What can be said about this double sum?

Question. Can this number be expressed in terms of classical values? $$\sum_{n,m=1}^{\infty}\frac1{(n^2+m^2)^{\frac32}}=1.056348517615643291\dots$$ UPDATE. I'm encouraged by Noam, Kevin and Igor's ...
T. Amdeberhan's user avatar
1 vote
1 answer
316 views

An explicit value for a bound proof

I saw a proof that $|p_n - li^{-1}(n)| \leq n e^{-c \sqrt{\ln(n)}} $, without saying anything about $c$ ! My questions is, what the explicit value of $c$ ?? It just says for some number $c$ without ...
Ahmad's user avatar
  • 595
3 votes
0 answers
202 views

Sum of multiplicative arithmetic function over squarefree numbers

In the "Sieve methods" notes of Dimitris Koukoulopoulos (see http://www.dms.umontreal.ca/~koukoulo/documents/notes/sievemethods.pdf), the following useful result can be found: Theorem 0.4.1. Let $g$ ...
user avatar
1 vote
2 answers
250 views

Enquiry on primorial numbers and primes

Does the inequality $$1 + \dfrac{\log p_{k+1}}{\log N_k} > (\log N_k)^{\dfrac{1}{p_{k+1} -1}}$$ hold for all integers $k\geq 2$, where $N_k$ denotes the $k$-th primorial number $N_k=p_1p_2\cdots ...
Community user.'s user avatar
2 votes
2 answers
725 views

Occurrence of simultaneous small remainders?

Fix $(a,b)=1$, $a<b<2a$ and $a,b>n^{1/(2t)}$ and fix a prime $T\approx n^{\tau+\frac1k}$ where $\tau\geq1$ and $k=2(t-1)$. We can show using exponential sums there is an $m_{_T}$ such that $T/...
user avatar
1 vote
1 answer
336 views

Question on the zeta and sigma functions

EDIT: The observation described here was due to a bug in the code used to generate the data (as commented by Lucia), which renders this question irrelevant. The answer, however, is worth reading. The ...
Maclio's user avatar
  • 37
1 vote
0 answers
192 views

Estimation of the $k$-th derivative zeta function

When I was doing some task in number theory which involves bounds for the Riemann zeta function, I was stuck on the following question: Let $\zeta$ be the Riemann zeta function and let $\zeta^{(k)}$ ...
Khadija Mbarki's user avatar
1 vote
1 answer
655 views

Could the complex zeros of Riemann zeta function be of the form $ s=0.5+ik$ with $k$ a positive integer? [closed]

I have checked in Andrew Odlyzko, Tables of zeros of the Riemann zeta function, to know if there is an example of zeros of Riemann zeta function with integer imaginary parts, but I don't see that. I ...
zeraoulia rafik's user avatar
4 votes
1 answer
491 views

An Exponential Sum Restricted to Primes

Let $a,q,N$ be integers such that $N/2 \leq q \leq N$ and $a/q \notin \mathbb{Z}$. Is the following estimate true, and, if so, how can it be proved? \[\left|\sum_{1 \leq p \leq N} \exp(2\pi i p a/q) \...
Linden's user avatar
  • 217
8 votes
1 answer
564 views

$\sum_{d\leq x} (\mu(d)/d) \log(x/d)$: is (the analogue of) Mertens' conjecture still false?

It is known to be false that $\sum_{m\leq x} \mu(m) \leq \sqrt{x}$ for all $x$ (Mertens' conjecture), and it is generally believed that $\lim \sup_{x\to \infty} |M(x)|/\sqrt{x} = \infty$. From the ...
H A Helfgott's user avatar
  • 19.4k
7 votes
2 answers
839 views

Closed formula for sine powers

I am looking for a closed formula for the expressions $$ \sum_{k=1}^{n-1} \sin\left(\pi \frac{k}{n}\right)^m,$$ with $n \in \mathbb{N}$ and $m \in \mathbb{N}$ odd. Playing with these sums a bit, I ...
Matthias Ludewig's user avatar
0 votes
2 answers
217 views

Sum of $\sum_{\substack{1<a<q \\(a,q)>1 \\ (a+1,q)>1}}1$

Let $a,q$ be a positive integers. I am trying to evaluate the following sum: $\sum_{\substack{1<a<q \\(a,q)>1 \\ (a+1,q)>1}}1$. Is there a formula that exists to calculate such sums? Here ...
usere5225321's user avatar
4 votes
1 answer
545 views

$\sum_{d\leq x} (\mu(d)/d) \log x/d$: elementary estimates?

Let $$F(x) = \sum_{d\leq x} \frac{\mu(d)}{d} \log \frac{x}{d}.$$ s it possible/feasible to give an elementary proof of the fact that $F(x)= 1 + o(1)$ (and, ideally, $1+O(1/\log x)$, or better)? By "...
H A Helfgott's user avatar
  • 19.4k
4 votes
0 answers
285 views

Dirichlet series of a lattice $\sum_{a \in \Lambda^*} |\det(a)|^{-s}$

For a lattice $\Lambda$ of rank $n$ in $\mathbb{Q}^{n\times n}$ whose non-zero elements are inversible matrices, let $$Z(s,\Lambda) = \sum_{a \in \Lambda^*} |\det(a)|^{-s}$$ I wonder if (and how to ...
reuns's user avatar
  • 3,405
2 votes
0 answers
545 views

Integrating a product of integrals involving Bessel functions

I have asked similar questions on Math Stack Exchange, but not been able to receive many helpful responses. Therefore, I am posting this problem here, and any input would be extremely valuable. I ...
user363087's user avatar
1 vote
1 answer
804 views

$n$th prime: a better approximation

Let $p_n$ be the $n$-th prime, then from Wikipedia I got that $p_n \approx n \left(\ln n + \ln \ln n -1 + \frac{\ln \ln n-2}{\ln n}+\frac{6\ln \ln n-( \ln \ln n)^2-11}{\ln^2 n} \right)$. What is a ...
Ahmad's user avatar
  • 595
2 votes
2 answers
352 views

Link between Irreducible Factors and Prime Factors (or Cycles of a Permutation)

In "Anatomy of Integers and Permutations", http://www.dms.umontreal.ca/~andrew/PDF/Anatomy.pdf, Granville gives a calibration of cycles of a permutation and prime factors of an integer. "We know ...
The Substitute's user avatar
7 votes
2 answers
804 views

On the consistency of the definition of the conductor for automorphic forms

Let $\pi$ be an irreducible admissible representation of $\mathrm{GL}_2(F)$, where $F$ is local non-archimedean. The local conductor associated to $\pi$ can be defined in two usual manners: By its ...
Desiderius Severus's user avatar
2 votes
1 answer
285 views

Exponential sum (linear in the argument) over primes

Suppose we have $\alpha \in \mathbb{R}$. Then we know that $$\sum_{1 \leq n \leq X} e(n \alpha) \ll \min \{ X, \|\alpha\|^{-1} \}$$ where $\| \cdot \|$ is the distance to the nearest integer. I ...
Johnny T.'s user avatar
  • 3,595
4 votes
1 answer
464 views

Odd Chebyshev, part 2

Let $$ \forall_{n=1\ 2\ \ldots}\quad I(n)\ :=\ \frac {(6\cdot n-3)!!}{(2\cdot n-1)!!\cdot(4\cdot n-3)!!} $$ Let $\ M(n)\ $ be the smallest natural number such that $$ M(n)\cdot I(n)\ \...
Włodzimierz Holsztyński's user avatar
31 votes
3 answers
7k views

Ideas in the elementary proof of the prime number theorem (Selberg / Erdős)

I'm reading the elementary proof of prime number theorem (Selberg / Erdős, around 1949). One key step is to prove that, with $\vartheta(x) = \sum_{p\leq x} \log p$, $$(1) \qquad\qquad \vartheta(x) \...
Basj's user avatar
  • 577
0 votes
1 answer
243 views

Can there be more than two zeta zeros in between a Gram point and a França-LeClair point?

According to formula 163 at page 47 in the paper A theory for the zeros of Riemann Zeta and other L-functions by Guilherme França and André LeClair, the Gram points can be approximated with the ...
Mats Granvik's user avatar
  • 1,133
6 votes
1 answer
728 views

Beauty of some numbers discovered by Ramanujan

I am a graduate PhD student and my topic is analytic number theory. I am also a mathematics teacher. I am planning to give a course to pupils in high school that motivates them to study arithmetic and ...
Khadija Mbarki's user avatar
5 votes
0 answers
187 views

Hilbert Modular Surface: Eigenfunctions of The Laplacian

The spectrum of the Laplacian on $L^2$ of a Hilbert modular surface decomposes into a discrete part and a continuous part $[1/4,\infty)$. The continuous part contains eigenvalues $\geq 1/4$. I would ...
espressionist's user avatar
0 votes
2 answers
385 views

What is the relationship between the abscissa of holomorphy and abscissa of convergence of a Dirichlet series

Given a Dirichlet series $$\phi(s)=\sum_{n\ge1}\frac{a_n}{n^s}$$ let $\sigma_{\text{conv}}\in\bar{\mathbb{R}}$ its abscissa of convergence, then we know that $\phi(s)$ is holomorphic on the half-plan $...
Adam's user avatar
  • 93
9 votes
2 answers
283 views

What is the density of integers of the form $a^2+nb^2$?

Landau proved that the mean density of integers of the form $a^2+b^2$ up to $x$ is $K\frac{x}{\sqrt{\log x}} (1+o(1))$, where $K$ is an explicit constant. One proof is based on the fact that a prime $...
Lior Bary-Soroker's user avatar
1 vote
0 answers
90 views

An arithmetic function involving arbitrary (but fixed) number of divisors

I need at least basic information about generating functions of the following class of arithmetic functions, grouped by levels $k$. Fix some $k$ and some family $\varepsilon_*=(\varepsilon_\sigma)_{\...
მამუკა ჯიბლაძე's user avatar
1 vote
0 answers
112 views

Zero mean constraint for correlations with the Mobius function

An aspect of my research has lead me to believe that I need to distinguish between those bounded functions $\xi:\mathbb{N}\rightarrow\mathbb{C}$ which correlate with the Mobius function $\mu(n)$, i.e. ...
Kevin Smith's user avatar
  • 2,470
0 votes
0 answers
160 views

Sequences that "capture their primes"

Let $g : \mathbb{N} \rightarrow \mathbb{N}$ be an increasing function, and consider the sequence $Y = \{y_n\}$ given by $y_n = g(n)$. Let $F$ be an irreducible binary form of degree $d \geq 2$ with ...
Stanley Yao Xiao's user avatar
1 vote
0 answers
192 views

Asymptotics for certain sum involving the divisor function, Ramanujan sum

Let $c_q(n)$ be the Ramanujan sum, and let $\tau(n)$ be the divisor function. Is there an asymptotic formula for $$\sum_{n\le x}\tau(n)c_q(n)$$ with error terms that do not depend on $q$?
Mayank Pandey's user avatar
3 votes
1 answer
354 views

A "nice" trigonometric polynomial approximation of a characteristic function

Let $\delta > 0$ be small and $\chi_{[-\delta, \delta]}(t)$ be a characteristic function on the interval $[-\delta, \delta]$. I am interested in a trigonometric polynomial $S$ such that $$ \chi_{[-\...
Johnny T.'s user avatar
  • 3,595

1
37 38
39
40 41
59